Acc 301 final

अब Quizwiz के साथ अपने होमवर्क और परीक्षाओं को एस करें!

Current asset examples

-Cash, prepaid rent for the next 9 months, inventories

15. Disclosure notes include:

-Depreciation methods used and estimated useful life. -Definition of cash equivalents. -Details of pension plans.

Investment and funds example

-Land, held for investment, investment in XYZ Corp, long term

Cash equivalent

is One-month U.S. Treasury bill.

Going concern assumption

the entity will continue indefinitely.

16.) The Mateo Corporation's inventory at December 31, 2018, was $325,000 based on a physical count priced at cost, and before any necessary adjustment for the following: Merchandise costing $30,000, shipped f.o.b. shipping point from a vendor on December 30, 2018, was received on January 5, 2019.Merchandise costing $22,000, shipped f.o.b. destination from a vendor on December 28, 2018, was received on January 3, 2019.Merchandise costing $38,000 was shipped to a customer f.o.b. destination on December 28, arrived at the customer's location on January 6, 2019.Merchandise costing $12,000 was being held on consignment by Traynor Company. What amount should Mateo Corporation report as inventory in its December 31, 2018, balance sheet?

$325,000 + $30,000 + $38,000 + $12,000 = $405,000

Liquidity refers to 17. Assets include

-the readiness of an asset to be converted to cash Paid in capital

Alamos Co. exchanged equipment and $18,000 cash for similar equipment. The book value and the fair value of the old equipment were $82,000 and $90,000, respectively.Assuming that the exchange lacks commercial substance, Alamos would record a gain/(loss) o

0

25.) Fulbright Corp. uses the periodic inventory system. During its first year of operations, Fulbright made the following purchases (listed in chronological order of acquisition): 40 units at $100 70 units at $80 170 units at $60 Sales for the year totaled 270 units, leaving 10 units on hand at the end of the year. Ending inventory using the FIFO method is:

10 units * $60= $600

13.) On January 1, Greenview Company adopted the dollar-value LIFO method. The inventory cost on January 1 was $112,000. On December 31, ending inventory had a cost of $136,400.

136,400/ 1.10=124,000 124,000-112,000=12,000 12,000 *1.10=13,200 112,000+13,200=125,200

12. Cal Farms reported supplies expense of $2,000,000 this year. The supplies account decreased by $200,000 during the year to an ending balance of $400,000. What was the cost of supplies the Cal Farms purchased during the year

400,000+2,000,000-600,000=1,800,000

5 21.Jose wants to cash in his winning lottery ticket. He can either receive five $5,000 annual payments starting today, or he can receive one lump-sum payment today based on a 3% annual interest rate. What would be the lump-sum payment?

5000*(1-(1+.03)^-5)/.03= 22899 22899*.03=687 22899+687= 23,586

The adjusting entry required when amounts previously recorded as deferred revenues are recognized includes

A debit to a liability.

Which of the following types of subsequent expenditures normally is capitalized?

Additions. Improvements. Rearrangements.

20. The following transactions occurred during March 2018 for the Wainwright Corporation. The company owns and operates a wholesale warehouse. A.) Issued 30,000 shares of common stock in exchange for $300,000 in cash. B.) Purchased equipment at a cost of $40,000. $10,000 cash was paid and a note payable was signed for the balance owed. C.) Purchased inventory on account at a cost of $90,000. The company uses the perpetual inventory system. D.) Credit sales for the month totaled $120,000. The cost of the goods sold was $70,000. E.) Paid $5,000 in rent on the warehouse building for the month of March. F.) Paid $6,000 to an insurance company for fire and liability insurance for a one-year period beginning April 1, 2018. G.) Paid $70,000 on account for the merchandise purchased in 3. H.) Collected $55,000 from customers on account. I.) Recorded depreciation expense of $1,000 for the month on the equipment Required:Analyze each transaction and show the effect of each on the accounting equation for a corporation.

CH2 put 20-22 on paper

CHAPTER 8

CHAPTER 8

21.Cash equivalents would not include:

Cash not available for current operations

The acquisition costs of property, plant, and equipment do not include:

Maintenance costs during the first 30 days of use.

material

Material is an aspect of relevance that depends on a company particular situation and is based on the nature or magnitude of the item that is being reported.

Intangible asset example

allowance for uncollectible accounts, patent

Predictive value and/or confirmatory value

is financial reporting means that information must be process Predictive value and/or confirmatory or both.

intangible asset

lack physical substance

T-accounts

used for instructional purposes instead of formal ledger accounts

Given identical current amounts owed and identical interest rates, annual payments of an ordinary annuity will be greater than annual payments of an annuity due.

True

An asset's residual value is the amount the company expects to receive for the asset at the end of its service life, less any anticipated disposal costs.

True

For companies that use FIFO or average cost, inventory is valued at the lower of cost or net realizable value at the end of the reporting period.

True

Gains or losses result, respectively, from the disposition of business assets for greater than, or less than, their book values. True or false

True

Revenues are inflows or other enhancements of assets or settlements of liabilities from activities that constitute the entity's ongoing operations. True or False

False

Sellers recognize revenue for gift cards at the point in time control of the gift card is transferred to the customer.

False

Sellers should recognize revenue over time for a long term contract in which the seller is receiving periodic payments for progress to date but may need to refund those payments in the event the contract is cancelled.

False

The "income statement approach" and the "direct write-off method" are two ways to refer to the same method for recognizing bad debts expense.

False

The FASB is currently the public-sector organization responsible for setting accounting standards in the United States True or False

False

The FASB's conceptual framework lists relevance and timeliness as the two fundamental qualitative characteristics of decision-useful information.( true or False)

False

The calculation of future value requires the removal of interest.

False

According to the FASB's Statements of Financial Accounting Concepts, conservatism is a desired qualitative characteristic of accounting information True or False

False

Accounting for stock-based compensation is an area in which the FASB has received little political interference. True or False

False

Accrual accounting attempts to measure revenues and expenses that occurred during accounting periods so they equal net operating cash flow. True or False

False

An asset's book value is computed as its original cost minus residual value, less accumulated depreciation.

False

Cash equivalents would include investments in marketable equity securities as long as management intends to sell the securities in the next three months

False

Companies always recognize revenue when goods or services are transferred to customers for the amount the company expects to receive in exchange for those goods or services.

False

Comprehensive income is another term for net income true or false

False

Demolition costs to remove an old building from land purchased as a site for a new building are considered part of the cost of the new building.

False

Determining fair value by calculating the present value of future cash flows is a level 1 type of input. True or False

False

Disclosure notes to the financial statements regarding significant revenue recognition policies are only required when they will not reveal important information to competitors, suppliers or customers.

False

Dollar-value LIFO eliminates the risk of LIFO liquidations.

False

Goods or services are distinct if they are either capable of being distinct or are separately identifiable

False

If a license is acquired to use intellectual property for a 5-year period, revenue always is recognized at the point in time the customer begins to benefit from the license

False

If a seller makes payments to a customer to purchase goods or services, and those payments are equal to the stand-alone selling prices of those goods or services, part of those payments are a refund to the customer.

False

If the estimate of a transaction price is revised, the price change is allocated entirely to the remaining performance obligations that are yet to be satisfied.

False

If the seller is an agent, the seller typically recognizes cost associated with the sale on its own line in the income statement.

False

In franchise arrangements, the franchisor's performance obligations are not separately identifiable, so revenue must be recognized over time.

False

Other things being equal, the present value of an annuity due will be less than the present value of an ordinary annuity.

False

Property, plant, and equipment and finite-life intangible assets must be tested for impairment at least once a year.

False

Revenue always is recognized once the buyer has physical possession of goods.

False

The periodicity assumption requires that present value calculations take into account the number of compounding periods in each year. True or False

False

The probability that the customer will pay the seller does not affect whether a contract exists for purposes of revenue recognition.

False

The right of return is a separate performance obligation, and a portion of the transaction price needs to be allocated to it for revenue recognition

False

The separate entity assumption states that, in the absence of contrary evidence, all entities will survive indefinitely. True or false

False

When a contract includes variable consideration, the probability-weighted amount must be used when there are different probabilities of occurrence.

False

When a long-term contract does not qualify for revenue recognition over time, all gross profit and loss recognition occurs when the contract is completed.

False

In computing capitalized interest, average accumulated expenditures

Is determined by time-weighting individual expenditures made during the asset construction period.

5. A payment on account has no effect on working capital but will increase the current ratio if it is already greater than 1.0. ( true or false)

True

5. Adjusting journal entries are recorded at the end of any period when financial statements are prepared.( True or False)

True

6. Return on shareholders' equity is increased if a firm can maintain its return on assets but increase its leverage.( True or False)

True

6.All current assets are either cash or assets that will be converted into cash or consumed within 12 months or the operating cycle, whichever is longer.( true or false)

True

7. Income from continuing operations sometimes includes gains from nonoperating activities.( True or false)

True

7. The balance of net receivables represents the amount expected to be collected.( True or false)

True

8. Prepaid expenses are classified as current assets if the services purchased are expected to expire within 12 months or the operating cycle, whichever is longer.(true or false

True

9. The statement of cash flows summarizes transactions that caused cash to change during a reporting period.( True or False)

True

A contract between a seller and a buyer need not be in writing to be enforceable.

True

A distinguishing characteristic of intangible assets is that the extent and timing of their future benefits typically are highly uncertain.

True

A goodwill impairment loss is indicated when the fair value of the reporting unit is less than its book value.

True

A license to use a company trademark should be viewed as an access right, with revenue recognized over the license period.

True

Auditors play an important role in the resource allocation process by adding credibility to financial statements. True or False

True

Compound interest includes interest earned on interest.

True

Equity is a residual amount representing the owner's interest in the assets of the business. True or false

True

If the seller is a principal, the seller typically is vulnerable to risks associated with returns of inventory from the customer

True

The calculation of present value eliminates interest from future cash flows.

true

The choice of cost flow assumption (FIFO, LIFO, or average) does not depend on the actual physical flow of the product.

true

The cost-to-retail percentage used in the retail method to approximate average cost incorporates both markdowns and markups.

true

Balance sheet

An organized array of assets, liabilities, and equity.

When using the gross profit method to estimate ending inventory, it is not necessary to know:

Cost of good slod

Time value of money

Money can be invested today and grow to a larger amount

Future value of a single amount

The amount of money that a dollar will grow to

On September 30, 2018, Corso Steel acquired a patent from Thermo Steel. The agreement specified that Corso will pay Thermo $1,000,000 immediately and then another $1,000,000 on September 30, 2020. An interest rate of 8% reflects the time value of money for this type of loan agreement. (PV of $1, PVA of $1) (Use appropriate factor(s) from the tables provided.) Corso should record the acquisition of the patent on September 30, 2018, for what amount?

$1,000,000 + ($1,000,000 × 0.85734*) = $1,857,340. (1+.08)^2= .85734 2020-2018= 2

35,) The Kwok Company's inventory balance on December 31, 2021, was $165,000 (based on a 12/31/2021 physical count) before considering the following transactions: Goods shipped to Kwok f.o.b. destination on December 20, 2021, were received on January 4, 2022. The invoice cost was $30,000.Goods shipped to Kwok f.o.b. shipping point on December 28, 2021, were received on January 5, 2022. The invoice cost was $17,000. Goods shipped from Kwok to a customer f.o.b. destination on December 27, 2021, were received by the customer on January 3, 2022. The sales price was $40,000 and the merchandise cost $22,000. Goods shipped from Kwok to a customer f.o.b. destination on December 26, 2021, were received by the customer on December 30, 2021. The sales price was $20,000 and the merchandise cost $13,000.Goods shipped from Kwok to a customer f.o.b. shipping point on December 28, 2021, were received by the customer on January 4, 2022. The sales price was $25,000 and the merchandise cost $12,000.Required:Determine the correct inventory amount to be reported in Kwok's 2021 balance sheet.

$165,000+ 17,000 +22,000= $204,000

44.)Mary signed up and paid $1200 for a 6 month ceramics course on June 1st with Choplet Ceramics. As of August 1st, Choplet's accounting records would indicate

$400 of revenue, $800 of deferred revenue ($1,200 × 2/6 of the contract duration) =400 revenue and deferred revenue of $800 ($1,200 contract price paid in advance - $400 revenue recognized to date).

19.) Alison's dress shop buys dresses from McGuire Manufacturing. Alison purchased dresses from McGuire on July 17 and received an invoice with a list price amount of $6,000 and payment terms of 2/10, n/30. Alison uses the net method to record purchases. Alison should record the purchase at:

$6,000 × 98%(2/10 is 2%) = $5,880 chapter 8

Holiday Laboratories purchased a high-speed industrial centrifuge at a cost of $420,000. Shipping costs totaled $15,000. Foundation work to house the centrifuge cost $8,000. An additional water line had to be run to the equipment at a cost of $3,000. Labor and testing costs totaled $6,000. Materials used up in testing cost $3,000. The capitalized cost is:

( 420,000+ 15,000+8,000+3,000+6,000+3,000)= 455,000

9.12 On July 8, a fire destroyed the entire merchandise inventory on hand of Larrenaga Wholesale Corporation. The following information is available: Sales, January 1 through July 8 $700,000 Inventory, January 1 130,000 Purchases, January 1 through July 8 640,000 Gross profit ratio 30% What is the estimated inventory on July 8 immediately prior to the fire?

($130,000+640,000)= 770,000 700,000* %30= 210,000 700,000-210,000= (490,000) 770,000- 490,000= 280,000

Determine how much will be accumulated in the fund on December 31, 2024 after four years, under each of the following situations. 1. The first $2,000 annual deposit is made at the end of each of the four years on December 31, 2021, and interest is compounded annually.2. The first $2,000 annual deposit is made at the beginning of each of the four years on December 31, 2020, and interest is compounded annually.3. The first $2,000 annual deposit is made at the beginning of each of the four years on December 31, 2020, and interest is compounded quarterly.4. The first $2,000 annual deposit is made at the beginning of each of the four years on December 31, 2020, interest is compounded annually, and interest earned is withdrawn at the end of each year.

(1.) FVA= (1+i)^n-1/i= $9559 n=4 i=%12 (2.) FVAD=[ (1+i)^n-1/i]*(1+i) =10,706 Deposit $2000 n=4 i=12% (3.)i= interest rate 3% n= quarterly = 4 2020 n=16 2021 n=12 2022 n= deposit 2,000 fund balance = (4.) Deposit amount 2000 No of paymenst=4 interest left in fund=o Fund balance= 8000

Grab Manufacturing Co. purchased a 10-ton draw press at a cost of $180,000 with terms of 5/15, n/45. Payment was made within the discount period. Shipping costs were $4,600, which included $200 for insurance in transit. Installation costs totaled $12,000, which included $4,000 for taking out a section of a wall and rebuilding it because the press was too large for the doorway. The capitalized cost of the 10-ton draw press is:

(180,000* 95%)= 171,000 171,000+ 4,600+12,000= 187,600

Cedric Company recently traded in an older model of equipment for a new model. The old model's book value was $180,000 (original cost of $400,000 less $220,000 in accumulated depreciation) and its fair value was $200,000. Cedric paid $60,000 to complete the exchange which has commercial substance. Required:Prepare the journal entry to record the exchange. (If no entry is required for a transaction/event, select "No journal entry required" in the first account field.)

(200,000+60,000)=260,000 gain on exchange (200,000-180,000)= 20,000 Equimpent-new 260,00dr Accumlated depreciation 220,00dr equimpent- old 400,000cr Cash 60,000cr gain on exchange of assset 20,000

10.) Otis Corp. uses a periodic system and the FIFO method. Otis had beginning inventory of 30 units purchased at $120 each and made the following purchases during the year:Jan. 15: 34 units at $110May 30: 61 units at $84Oct. 20: 160 units at $60Sales during the year totaled 271 units. What is the cost of ending inventory? 840

(30 + 34 + 61 + 160 − 271) are from the last purchase14 units × $60 = $840

11.) Carrington Corp. uses a periodic system and the LIFO method. Carrington had beginning inventory of 30 units purchased at $120 each and made the following purchases during the year:Jan. 15: 34 units at $110May 30: 61 units at $84Oct. 20: 160 units at $60Sales during the year totaled 271 units. What is the cost of ending inventory?Cost of ending inventory

(30 + 34 + 61 + 160 − 271) are from the beginning inventory.14 units × $120 = $1,680

Granite Enterprises acquired a patent from Southern Research Corporation on January 1, 2021, for $4 million. The patent will be used for five years, even though its legal life is 20 years. Rocky Corporation has made a commitment to purchase the patent from Granite for $200,000 at the end of five years. Compute Granite's patent amortization for 2021, assuming the straight-line method is used

(4,000,000-200,000)/5= 760,000

10.3 Mendelson Laboratories purchased engineering equipment at a cost of $420,000. Shipping costs totaled $15,000. Installation cost was $8,000. An additional electrical line had to be run to the equipment at a cost of $3,000. Labor and testing costs totaled $6,000. Materials used up in testing cost $3,000. The capitalized cost is

(420,00 +15,000+ 8,000 + 3,000 6,000 + 3,000= 455,000) $455,000

14. Misty Company reported the following before-tax items during the current year: Sales revenue$600 Selling and administrative expenses 250 Restructuring charges 20 Loss on discontinued operations 50 Misty's effective tax rate is 40%. What is Misty's income from continuing operations?

(600-250-20) * (1- .04)= 198

3. Misty Company reported the following before-tax items during the current year: Sales revenue$600 Selling and administrative expenses 250 Restructuring charges 20 Loss on discontinued operations 50 Misty's effective tax rate is 40%.What is Misty's net income for the current year?

(600-250-20)= 330 (330* 40%)=132 (50-20)=30 330-132-30=168

Alamos Co. exchanged equipment and $18,000 cash for similar equipment. The book value and the fair value of the old equipment were $82,000 and $90,000, respectively. Assuming that the exchange has commercial substance, Alamos would record a gain/(loss) of:

(90,000-82,000)=8,000

Which of the following statements about bill-and-hold arrangements are true?

-A bill-and-hold arrangement occurs when a customer purchases goods but requests that shipment occur at a later date. -In most bill-and-hold arrangements, sellers usually conclude that revenue should not be recognized until actual delivery to the customer occurs. -SEC guidance regarding bill-and-hold arrangements include the requirement that the product must be specifically identified as belonging to the customer and is ready for shipment.

cash equilvent

-Cash in a checking account ( included)-U.S. Treasury bill that mature in six months (Excluded) -Undeposited customer checks ( Included) -Cash in a saving account(Included) - currency and coins ( included)

Hales Inc. ships goods on December 30 to Osher Inc., who receives the goods on January 2. Both companies have December 31 year-ends. If the goods are shipped f.o.b. shipping point, which of the following statements is (are) correct?

-Osher will include the goods in its December 31 inventory. Osher will recird the purchase on december 30Hales will record the sale on December 30.

Which of the following statements about the types of inventory is (are) correct?Merchandising companies produce the goods they sell to customers.Merchandising companies produce the goods they sell to customers. CorrectWhen the manufacturing process is complete, the cost of the related inventory items is transferred into finished goods.When the manufacturing process is complete, the cost of the related inventory items is transferred into finished goods. CorrectInventory is classified as an asset in the balance sheet until it is sold, at which time the cost is transferred to cost of goods sold in the income statement.Inventory is classified as an asset in the balance sheet until it is sold, at which time the cost is transferred to cost of goods sold in the income statement. CorrectCosts necessary to get inventory in condition and location for sale are not included as a cost of inventory.Costs necessary to get inventory in condition and location for sale are not included as a cost of inventory.

-When the manufacturing process is complete, the cost of the related inventory items is transferred into finished goods. -Inventory is classified as an asset in the balance sheet until it is sold, at which time the cost is transferred to cost of goods sold in the income statement.

3. Balance sheet accounts are referred to as temporary accounts because their balances are always changing. ( true or False)

False

Components(1-5)

1. Cash and cash equivalents 2. Short term investments 3. Account receivable 4. Inventory 5. Prepaid expense

22.The following transactions occurred during December 31, 2018, for the Falwell Company. A three-year fire insurance policy was purchased on July 1, 2018, for $12,000. The company debited insurance expense for the entire amount. Depreciation on equipment totaled $15,000 for the year. Employee salaries of $18,000 for the month of December will be paid in early January 2019. On November 1, 2018, the company borrowed $200,000 from a bank. The note requires principal and interest at 12% to be paid on April 30, 2019. On December 1, 2018, the company received $3,000 in cash from another company that is renting office space in Falwell's building. The payment, representing rent for December and January, was credited to deferred rent revenue.

1. Prepaid Insurance 10,000dr Insurance expense 10,000cr 12,000*30/36=10,000 2. Depreciation expense 15,000dr Acumlated depreciation 15,000cr 3. Salaries expense 18,000dr Salaries payable 18,000cr 4.)Interest expense 4,000cr Interest payble 4,000dr 200,000*.12* 2/12 5.) Deferred rent revenue 1,500cr Rent revenue 1,500dr 1/2* 3,000=1500

Place the steps at the end of the accounting period in the correct order

1. Prepare an unadjusted trial balance 2. Record adjusting entries 3. Prepare an adjusted trial balances 4. Prepare finical statements Close the temporary accounts

1.Investing activities 2.Operating activities 3.Operating activities 4.Financing activities 5.Investing activities 6.Financing activities 7.Investing activities 8.Operating activities 9.Noncash investing and financing 10.Financing activities 11.Financing activities

1. Purchase of equipment for cash. 2. Payment of employee salaries. 3. Collection of cash for customers. 4. Cash proceeds from a note payable. 5.Purchase of common stock of another corporation for cash. Issuance of common stock for cash. 7. Sale of equipment for cash. 8. Payment of interest on note payable. 9. Issuance of bonds payable in exchange for land and building. 10. Payment of cash dividends to shareholders. 11. Payment of principal on note payable.

ch 2 21.The following transactions occurred during March 2018 for the Wainwright Corporation. The company owns and operates a wholesale warehouse. 1. Issued 30,000 shares of non-par common stock in exchange for $300,000 in cash. 2. Purchased equipment at a cost of $40,000. $10,000 cash was paid and a note payable was signed for the balance owed. 3. Purchased inventory on account at a cost of $90,000. The company uses the perpetual inventory system. 4. Credit sales for the month totaled $120,000. The cost of the goods sold was $70,000. 5. Paid $5,000 in rent on the warehouse building for the month of March. 6. Paid $6,000 to an insurance company for fire and liability insurance for a one-year period beginning April 1, 2018. 7.) Paid $70,000 on account for the merchandise purchased in 3. 8.) Collected $55,000 from customers on account. 9.) Recorded depreciation expense of $1,000 for the month on the equipment.

1.) Cash 300,000dr Commonstock 300,000dr 2.) Equimpent 40,000 dr Cash 10,000cr Note Payable 30,000cr 3.) inventory 90,000dr Account payable 90,000cr 4.a) account receivable 120,000 dr sale revenue 120,000cr 4b) COGS 70,000dr inventory70,00 cr 5.) `Rent expense 5,000dr Cash 5,000cr 6. Prepaid insurance 6,000dr cash 6,000cr 7.account payable 70,000dr cash 70,000 cr 8. cash 55,000dr account receivable 55,000cr 9.depreciation expense 1.000dr accumulated depreciation 1,000cr

23.) Fulbright Corp. uses the periodic inventory system. During its first year of operations, Fulbright made the following purchases (listed in chronological order of acquisition) :40 units at $100 70 units at $80 170 units at $60 Sales for the year totaled 270 units, leaving 10 units on hand at the end of the year. Ending inventory using the LIFO method is:

10 units * $100= 1,000

11. On August 1, 2018, Rocket Retailers adopted a plan to discontinue its catalog sales division, which qualifies as a separate component of the business according to GAAP regarding discontinued operations. The disposal of the division was expected to be concluded by June 30, 2019. On January 31, 2019, Rocket's fiscal year-end, the following information relative to the discontinued division was accumulated: Operating loss Feb. 1, 2018-Jan. 31, 2019 $115,000 Estimated operating losses, Feb. 1-June 30, 2019 80,000 Impairment of division assets at Jan. 31, 2019 10,000 In its income statement for the year ended January 31, 2019, Rocket would report a before-tax loss on discontinued operations of:

115,000+10,000=125,000

Below is information relative to an exchange of similar assets by Grand Forks Corp. Assume the exchange has commercial substance. Cash Book /Value Fair Value /Paid Case A$50,000 $60,000 $15,000 Case B$40,000 $35,000 $8,000 In Case B, Grand Forks would record a gain/(loss) of:

40,00-35,000= (5,000)

19. December 31 is as follows: Prepaid insurance at January 1 $52,500 Insurance expense recognized during the year 218,750 Prepaid insurance at December 31 61,250 What was the total amount of cash paid by Castle for insurance premiums during the year?

218,750-52,500+61,250= 227,500

10. 35 On January 1, 2018, Kendall Inc. began construction of an automated cattle feeder system. The system was finished and ready for use on September 30, 2019. Expenditures on the project were as follows: January 1, 2018 $200,000 September 1, 2018 $300,000 December 31, 2018 $300,000 March 31, 2019 $300,000 September 30, 2019$200,000 Kendall borrowed $750,000 on a construction loan at 12% interest on January 1, 2018. This loan was outstanding throughout the construction period. The company had $4,500,000 in 9% bonds payable outstanding in 2018 and 2019. Average accumulated expenditures for 2018 was:

200 *12/12= 200,000 300,000* 4/12= 100,000 300,000* 0/12= 0 200,000+100,000= 300,000

On January 1, 2018, Kendall Inc. began construction of an automated cattle feeder system. The system was finished and ready for use on September 30, 2019. Expenditures on the project were as follows: January 1, 2018 $200,000 September 1, 2018 $300,000 December 31, 2018 $300,000 March 31, 2019 $300,000 September 30, 2019 $200,000 Kendall borrowed $750,000 on a construction loan at 12% interest on January 1, 2018. This loan was outstanding throughout the construction period. The company had $4,500,000 in 9% bonds payable outstanding in 2018 and 2019. Interest capitalized for 2018 was:

200 *12/12= 200,000 300,000* 4/12= 100,000 300,000* 0/12= 0 200,000+100,000= 300,000 300,000* .12= 36,000

On January 1, 2021, Tabitha Designs purchased a patent for $240,000 giving it exclusive rights to manufacture a new type of synthetic clothing. While the patent had a remaining legal life of 15 years at the time of purchase, Tabitha expects the useful life to be only eight more years. In addition, Tabitha purchased equipment related to production of the new clothing for $140,000. The equipment has a physical life of 10 years, but Tabitha plans to use the equipment only over the patent's service life and then sell it for an estimated $20,000. Tabitha uses straight-line for all long-term assets. The amount to expense in 2024 related to the patent and equipment should be:

240,000/8= 30,000 140,000-20,000/8= 15,000 30,000+15,000= 45,000

Pro-tech Software acquired all of the outstanding stock of Reliable Software for $14 million. The book value of Reliable's net assets (assets minus liabilities) was $8.3 million. The fair values of Reliable's assets and liabilities equaled their book values with the exception of certain intangible assets whose fair values exceeded book values by $2.5 million. Calculate the amount paid for goodwill. (Enter your answer in whole dollars.)

3,200,000 8,300,000+2,500,000= 10,800,000 14,00,000-10,800,000

9.15 Benny's Bed Co. uses a periodic inventory system and the average cost retail method to estimate ending inventory and cost of goods sold. The following data is available from the company records for the month of September 2018. Cost Retail Beginning inventory $30,000 $50,000 Net purchases 125,000 220,000 Net markups 15,000 (retail) Net markdowns 6,000 (retail) Net sales 208,000 (retail) The average cost-to-retail percentage (rounded) is:

30,000+125,000= 155,000 50,000+220,000+15,000- (6,000)= 279,000 155,000/279,000= 55.6%

Savings Mart is a national retail chain. To entice the company to open a mega store in its jurisdiction, the city of Populationville donated a 20-acre tract of land to be used for construction. The land was originally purchased by the city for $250,000 three years ago. The appraisal value at the time of the donation was $300,000. For what amount should Savings Mart record the donated land?

300,000

On January 1, 2018, Dreamworld Co. began construction of a new warehouse. The building was finished and ready for use on September 30, 2019. Expenditures on the project were as follows: January 1, 2018 $300,000 September 1, 2018 $450,000 December 31, 2018 $450,000 March 31, 2019 $450,000 September 30, 2019$300,000 Dreamworld had $5,000,000 in 12% bonds outstanding through both years. Dreamworld's average accumulated expenditures for 2018 was

300,000* 12/12=300,000 450,000*4/12= 150,000 450,000*0/12=0 450,000 sum

16.The employees of Neat Clothes work Monday through Friday. Every other Friday the company issues payroll checks totaling $32,000. The current pay period ends on Friday, July 3. Neat Clothes is now preparing quarterly financial statements for the three months ended June 30. What is the adjusting entry to record accrued salaries at the end of June?

32,000 * 7/10= 22,400 Salaries expense 22,400dr Salaries payable 22,400cr

9.23 Prunedale Co. uses a periodic inventory system. Beginning inventory on January 1 was overstated by $32,000, and its ending inventory on December 31 was understated by $62,000. These errors were not discovered until the next year. As a result, Prunedale's cost of goods sold for this year was:

32,000 + 62,000= 94,000 overstated

In 2018, Martin Corp. acquired Glynco and recorded goodwill of $100 million. Martin considers Glynco a separate reporting unit. By the end of 2021, the net assets (including goodwill) of Glynco are $320 million and its estimated fair value is $260 million. The amount of the impairment loss that Martin would record for goodwill at the end of 2021 is:

320 million - 260 million = 60 million

28.Data related to the inventories of Costco Medical Supply are presented below: Selling price$260 $100 $340 $165 Cost 170 90 250 162 Costs to sell 30 15 25 10 In applying the lower of cost or net realizable value rule, the inventory of rehab equipment would be valued at:

340-24= $315 250 is the answer is less than 315 net reliable value

On July 1, 2018, Larkin Co. purchased a $400,000 tract of land that is intended to be the site of a new office complex. Larkin incurred additional costs and realized salvage proceeds during 2018 as follows: Demolition of existing buildin$75,000 Legal and other fees to close 12,000 Proceeds from sale of 10,000 What would be the balance in the land account as of December 31, 2018?

400,000+75,000+12,000-10,000= 477,000

On April 17, 2021, the Loadstone Mining Company purchased the rights to a coal mine. The purchase price plus additional costs necessary to prepare the mine for extraction of the coal totaled $4,500,000. The company expects to extract 900,000 tons of coal during a four-year period. During 2021, 240,000 tons were extracted and sold immediately. Required:1. Calculate depletion for 2021.2. Is depletion considered part of the product cost and included in the cost of inventory?

4,500,000/900,000= $5.00 240,000* $5.00= 1,200,000 yes

9.17 Willie Nelson's Boots uses the conventional retail method to estimate ending inventory. Cost data for the most recent quarter is shown below: Cost Retail Beginning inventory$46,000 $63,000 Net purchases 154,000 215,000 Net markups 22,000 (retail) Net markdowns 35,000 (retail) Net sales 220,000 ( retail) The conventional cost-to-retail percentage (rounded) is:

46,000+154,000= 200,000 63,000+215,000+22,000= 300,00 200,000/300,000= 66.7 or 2/3

17.On September 1, 2018, Fortune Magazine sold 600 one-year subscriptions for $81 each. The total amount received was credited to deferred subscriptions revenue. What is the required adjusting entry at December 31, 2018

48,600* 4/12 Cash 48,600dr Deferred subscription revenue 48,6000cr

43.On November 1, 2018, Taylor signed a one-year contract to provide handyman services on an as-needed basis to King Associates, with the contract to start immediately. King agreed to pay Taylor $4,800 for the one-year period. Taylor is confident that King will pay that amount, but payment is not scheduled to occur until 2019. Taylor should recognize revenue in 2018 in the amount of

4800 * 2/12)= 8000

In 2021, a company purchased two patents. Related information follows: Patent 1 Patent 2 Purchase price$500,000 $200,000 Legal and filing fees 25,000 20,000 In addition, near the end of 2021, the legality of both patents was challenged. The company paid $50,000 to defend Patent 1, and the defense was successful. The company paid $40,000 to defend Patent 2, but the defense was not successful and the patent was determined to have no value. Ignoring any amortization in 2021, the amount of expenditures to capitalize at the end of the year is:

500,000+25,000+50,000= 575,000

Marilee's Electronics uses a periodic inventory system and the average cost retail method to estimate ending inventory and cost of goods sold. The following data is available from the company records for the month of June 2018: Cost Retail Beginning inventory$80,000 $130,000 Net purchases 261,000 500,000 Net markups 25,000 Net markdowns 35,000 Net sales 520,000 To the nearest thousand, estimated ending inventory is:

55,000

Pensacola Inc. exchanged old equipment for new equipment in two exchange transactions. Each transaction has commercial substance. Cash Book /Value Fair /Value Received Equipment A$75,000 $80,000 $12,000 Equipment B$60,000 $56,000 $10,000 For Equipment B, Pensacola would record a gain/(loss) of:

60,000-56,000= (4,000)

13.On November 1, 2018, Jamison Inc. adopted a plan to discontinue its barge division, which qualifies as a separate component of the business according to GAAP regarding discontinued operations. The disposal of the division was expected to be concluded by April 30, 2019. On December 31, 2018, the company's year-end, the following information relative to the discontinued division was accumulated: Operating loss Jan. 1-Dec. 31, 2018 $65million Estimated operating losses, Jan. 1 to April 30, 2019 80million Excess of fair value, less costs to sell, over book value at Dec. 31, 2018 15million In its income statement for the year ended December 31, 2018, Jamison would report a before-tax loss on discontinued operations of

65 million

22.) Inventory records for Herb's Chemicals revealed the following: March 1, 2018, inventory: 1,000 gallons @ $7.20 = $7,200 Purchases: Sales:Mar. 10 600gals @$7.25 Mar. 5 400galsMar. 16 800gals @$7.30 Mar. 14 700gals Mar. 23 600gals @$7.35 Mar. 20 500gals Mar. 26 700gals

700 × $7.20 = answ $5,040 (600gals +800 gals+ 00 gals = 2000 Gallons sold (400 + 700 + 500 + 700)= (2,300)1000+2000- 2300=700

Marilee's Electronics uses a periodic inventory system and the average cost retail method to estimate ending inventory and cost of goods sold. The following data is available from the company records for the month of June 2018: Cost Retail Beginning inventory $80,000 $130,000 Net purchases 261,000 500,000 Net markups 25,000 Net markdowns 35,000 Net sales 520,000 The average cost-to-retail percentage is:

80,000c+ 261,000c=341,000 130,000+500,000+25,000-35,000= 620,000 ($341000/$620000)*100= 55%

Lake Incorporated purchased all of the outstanding stock of Huron Company paying $950,000 cash. Lake assumed all of the liabilities of Huron. Book values and fair values of acquired assets and liabilities were: Book Value Fair Value Current assets (net)$130,000 $125,000 Property, plant, 600,000 750,000 Liabilities 150,000 175,000 Lake would record goodwill of:

875,000-175,000= 700,000 950,000-700,00=250,000

A rules-based approach to standard-setting stresses professional judgment as opposed to following a list of rules True or False

False

5 (16.) Ajax Company purchased a five-year certificate of deposit for its building fund in the amount of $220,000. How much should the certificate of deposit be worth at the end of five years if interest is compounded at an annual rate of 9%?

9%= .09 5yrs= ^5 220,000(1+.09)^5= 142,985

1. Material restructuring costs are reported as an element of income from continuing operations( True or False)

True

Which of the following statements about changes to the LIFO inventory method from any other method is true?

A disclosure note is needed.

Which of the following statements about franchises are true?

A franchise involves a license to use intellectual property .- In a franchise arrangement, a franchisor grants to the franchisee the right to sell the franchisor's products and use its name.

4c. In general, revenue is recognized when

A good or service has been delivered to a customer

16.Rent collected in advance is:

A liability account in the balance sheet.

A change from the straight-line method to the double-declining-balance method of depreciation is handled as:

A prospective change from the current year through the remainder of its useful life.

annuity

A series of equal-sized cash flows

Which of the following is not an indicator that the constraint on recognizing variable consideration should be applied? -Poor (limited) evidence on which to base an estimate -A broad range of outcomes that could occur -A short delay before uncertainty resolves -A history of the seller changing payment terms on similar contracts

A short delay before uncertainty resolves

4. After an unadjusted trial balance is prepared, the next step in the accounting processing cycle is the preparation of financial statements.( True or False)

False

A. Most license of functional intellectual property B. Most licenses pf symbolic intellectual property C. A license granted by an online service to customers to access content at a website

A. Seller recognize revenue at a point in time B. Seller recognize revenue over time C. Seller recognizes revenue over time

5 19. You want to invest $20,000 today to accumulate $32,000 for graduate school. If you can invest at an interest rate of 10% compounded annually. To find how many years will it take to accumulate the required amount, you would search the 10% column in the:

ANSW= Present value of $1 table, for the factor closets to .625 32,000/20,000=.625

Stayman Associates has sold a good to a buyer and wants to recognize revenue. Which of the following is an indicator that control of a good has passed from Stayman to the buyer?

Buyer has assumed the risk and rewards of ownership.

Oswego Clay Pipe Company sold $46,000 of pipe to Southeast Water District #45 on April 12 of the current year with terms 1/15, n/60. Oswego uses the gross method of accounting for cash discounts.What entry would Oswego make on April 12?

Accounts receivable46,000 Sales 46,000

Full Disclosure Principle

All information that could affect decisions should be reported.

interest

Amount of money paid/received in excess of amount borrowed/lent

(3.)John Rider wants to accumulate $100,000 to be used for his daughter's college education. He would like to have the amount available on December 31, 2026. Assume that the funds will accumulate in a certificate of deposit paying 8% interest compounded annually. (FV of $1, PV of $1, FVA of $1, PVA of $1, FVAD of $1 and PVAD of $1) (Use appropriate factor(s) from the tables provided.)Answer each of the following independent questions. Required: 1. If John were to deposit a single amount, how much would he have to invest on December 31, 2021? 2. If John were to make five equal deposits on each December 31, beginning a year later, on December 31, 2022, what is the required amount of each deposit?3. If John were to make five equal deposits on each December 31, beginning now, on December 31, 2021, what is the required amount of each deposit?

Amount= PV*(1+i)^n 100,000/(1+8%)^5= 68,058 .1.Single deposit, n per = 2018-2013 = 5 Yrs FV = PV*(1+i)^nS PV = FV/(1+i)^n = 100,000/(1+8%)^5 = $68,058SO 2. We have 5 Equal deposit = PMTSO we have Rate = 8%, n per = 5, FV=100,000, PMT(Rate , n per ,PV,FV)= PMT(8%,5,,100000) = $17,0463. 3.we have Rate = 8%, nper = 5, FV=100,000, PV= Annual Payments = PMT(Rate,nper,PV,FV,type)= PMT(8%,5,,100000,1) = $15,783

10, Sullivan Corporation has determined its year-end inventory on a FIFO basis to be $500,000. Information pertaining to that inventory is as follows: Selling price$520,000 Costs to sell 30,000 Replacement cost 440,000 What should be the reported value of Sullivan's inventory?

Answ. $490,000 NRV = $520,000 ‒ $30,000 = $490,000Net realizable value = $490,000 which is less than $500,000 cost.

9.9 Data related to the inventories of Costco Medical Supply are presented below: Surgical Surgical/ Rehab Rehab/ Equipment Supplies /Equipment Supplies Selling price$260 $100 $340 $165 Cost 170 90 250 162 Costs to sell 30 15 25 10 In applying the lower of cost or net realizable value rule, the inventory of surgical supplies would be valued at:

Answ. 85 NRV = $100 - 15 = $85$85 net realizable value is less than $90 cost.

4. The following balance sheet for the Los Gatos Corporation was prepared by a recently hired accountant. In reviewing the statement you notice several errors. LOS GATOS CORPORATIONBalance SheetAt December 31, 2018 Assets Cash$40,000 Accounts receivable 80,000 Inventories 55,000 Machinery (net) 120,000 Franchise (net) 30,000 Total assets$325,000 Liabilities and Shareholders' Equity Accounts payable$50,000 Allowance for uncollectible accounts 5,000 Note payable 55,000 Bonds payable 110,000 Shareholders' equity 105,000 Total liabilities and shareholders' equity$325,000 Additional information: 1.) Cash includes a $20,000 restricted amount to be used for repayment of the bonds payable in 2022. 2.) The cost of the machinery is $190,000. 3.)Accounts receivable includes a $20,000 note receivable from a customer due in 2021. 4.) The note payable includes accrued interest of $5,000. Principal and interest are both due on February 1, 2019. 5.) The company began operations in 2013. Income less dividends since inception of the company totals $35,000. 6.) 50,000 shares of no par common stock were issued in 2013. 100,000 shares are authorized. Required:Prepare a corrected, classified balance sheet. Enter amounts in order of liquidity.

Asset Current assets cash 20,000cr Account receivable (net) 55,000cr Inventories 55,000cr Total current assets 130,000 cr investments Restricted cash 20,000dr note receivable 20,000dr Total investment 40,000cr property,plant, and equipment Machinery 190,000dr Accumlated depreciation (70,000) Net property plant and equipment 120,000cr Intangible asset Franchise 30,000dr Total asset 320,000dr Liabilities and shareholders equity Current liabilities Account payable 50,000cr Interest payable 5,000cr Note payable 50,000cr Total curretr liabilities 105,000cr Long term liabilites bonds payable 110,00cr Total liabilites 215,000cr shareholder equity commonstock 70,000dr retained earnings 35,000dr total shareholders equity 105,000 TOTAL liabilites and shareholder 320,000cr

Which of the following is not an indicator that the customer is likely to have control over a good?- Asset warehoused by seller-affiliated third party -Accepted the asset -Legal title to the asset

Asset warehouse by seller-affiliated third party

In testing for recoverability of property, plant, and equipment, an impairment loss is required if the:

Asset's book value exceeds the undiscounted sum of expected future cash flows.

Where is management's discussion and analysis located?

At the end of the annual report as an appendix Default risk describes the risk the company cannot pay its obligations when they come due.

Retrospective treatment of prior years' financial statements is required when there is a change from:

Average cost to FIFO. FIFO to average cost. LIFO to average cost.

On September 22, 2021, a flood destroyed the entire merchandise inventory on hand in a warehouse owned by the Rocklin Sporting Goods Company. The following information is available from the records of the company's periodic inventory system: Inventory, January 1, 2021 $140,000 Net purchases, January 1 through September 22 370,000 Net sales, January 1 through September 22 550,000 Gross profit ratio 25% Required:Complete the below table to estimate the cost of inventory destroyed in the flood using the gross profit method.

Beginning inventory 140,000cr Plus net purchases 370,000cr cogs available for sale 510,000cr Less COGS net sale 550,000dr less estimated gross profit (137,500)dr estimated cogs 412,500 cr estimated cost of inventory destroyed 97,500cr

What is the relationship between Construction-in-progress (CIP) and the Billings on Construction Contract account?

Billings is contra to CIP and reduces the balance of the CIP account.

Bluestein Inc. purchases goods from one of its suppliers and incurs freight charges to have the goods delivered. The company uses a perpetual system. Which of the following statements is correct?

Bluestein will record the freight charges in its Inventory account.

5. A company could improve its return on assets by increasing its income or by increasing its total assets.( True or False)

False

12. Cal Farms reported supplies expense of $2,000,000 this year. The supplies account decreased by $200,000 during the year to an ending balance of $400,000. What was the cost of supplies the Cal Farms purchased during the year

Cal Farms reported supplies expense of $2,000,000 this year. The supplies account decreased by $200,000 during the year to an ending balance of $400,000. What was the cost of supplies the Cal Farms purchased during the year

2>The controller of the Red Wing Corporation is in the process of preparing the company's 2018 financial statements. She is trying to determine the correct balance of cash and cash equivalents to be reported as a current asset in the balance sheet. The following items are being considered: A.)Balances in the company's accounts at the First National Bank; checking $13,500, savings $22,100. B.) Undeposited customer checks of $5,200. C.) Currency and coins on hand of $580. D.) Savings account at the East Bay Bank with a balance of $400,000. This account is being used to accumulate cash for future plant expansion (in 2020) .E.) $20,000 in a checking account at the East Bay Bank. The balance in the account represents a 20% compensating balance for a $100,000 loan with the bank. Red Wing may not withdraw the funds until the loan is due in 2021. F.) S. Treasury bills; 2-month maturity bills totaling $15,000, and 7 month bills totaling $20,000.Required:1. Determine the correct balance of cash and cash equivalents to be reported in the current asset section of the 2018 balance sheet.

Cash and cash equivalent includes: A. -Balance in checking account $13,5000- Balance in saving account $22,100 B. Undeposited customer checks $5,200 C. Currency and coin on hand $580 D. Balance in saving account $0 E. Balance in checking account $0 F. U.S. treasury bill 15,000Total 58,380

13. Which of the following is not an adjusting entry -Prepaid event/ rent expense -Cash deferred/ revenue -Interest expense/ interest payable -Salaries expense/ salary payable

Cash deferred/ revenue

On November 10 of the current year, Flores Mills sold carpet to a customer for $8,000 with credit terms 2/10, n/30. Flores uses the gross method of accounting for cash discounts.What is the correct entry for Flores on November 17, assuming the correct payment was received on that date?

Cash7,840 dr Sales discounts160 dr Accounts receivable 8,000 cr

Chapter 5

ChAPTER 5

Chapter 10

Chapter 10

Chapter 2

Chapter 2

Chapter 3

Chapter 3

chapter 4

Chapter 4

Monetary asset

Claim to receive a fixed amount of money

Using the dollar-value LIFO retail method for inventory:

Combines retail LIFO accounting with dollar-value LIFO accounting.

simple interest

Computed by multiplying an invested amount by the interest rate

14. Management's Report on Internal Control Over Financial Reporting:

Contains personal certification of the financial statements by the company's executives.

6. Accruals occur when the cash flow precedes either revenue or expense recognition.( True or False)

False

7. The income statement summarizes the operating activity of a firm at a particular point in time.(True or False)

False

8. The balance sheet can be considered a change or flow statement.( True or False)

False

3.)Barrington Company began the year with inventory of $100,000. During the year, the company purchased inventory in the amount of $750,000. Sales revenue for the year totaled $800,000. A physical count determined the cost of inventory at the end of the year to be $90,000. The adjusting entry needed at the end of the year under a periodic inventory system includes a:

Debit to Cost of Goods Sold for $760,000 .answ. Beginning inventory+$100,000 Plus: Purchases +750,000 Cost of goods available for sale= 850,000 Less: Ending inventory -90,000 Cost of goods sold$760,000

Maltese is a privately-owned company. On September 3, Maltese exchanged 2,000 shares of its private common stock for equipment. There is no readily available estimate of the stock's fair value. The equipment currently is selling for $80,000. The journal entry to record this transaction includes:

Debit: Equipment, $80,000.

How are abnormal shortages handled when using the retail inventory method to estimate ending inventory?

Deducted in both the cost and retail columns before the calculation of the cost-to-retail percentage

Clark Oil and Gas incurred costs of $15.3 million for the rights to extract resources from a natural gas deposit. The company expects to extract 8 million cubic feet of natural gas during a six-year period. Natural gas extracted during years 1 and 2 were 800,000 and 1,600,000 cubic feet, respectively. What was total depletion for year 1 and year 2, assuming the company uses the units-of-production method?

Depletion rate = $15.3 million ÷ 8 million cubic feet = $1.9125/cubic foot. Total depletion = (800,000 + 1,600,000) × $1.9125 = $4.59 million.

The first step, when using dollar-value LIFO retail method for inventory, is to:

Determine the cost-to-retail percentage for the current year transactions.

Which of the following are situations where estimates of inventory, rather than a physical count of inventory, may be desirable? (Select all that apply.)

Determining the cost of inventory that has been lost, destroyed, or stolen Estimating inventory and cost of goods sold for interim financial reports Avoiding the expense of a physical inventory count In auditors' testing of the overall reasonableness of inventory amounts reported by clients In budgeting and forecasting

Loan A has the same original principal, interest rate, and payment amount as Loan B. However, Loan A is structured as an annuity due, while Loan B is structured as an ordinary annuity. The maturity date of Loan A will be

Earlier than loan B

When revenues, costs and gross profit are recognized at the completion of the contract rather than periodically throughout the contract:

Either method results in the same revenues, costs and gross profits being recognized by the end of the project.

On January 1, 2021, the Brunswick Hat Company adopted the dollar-value LIFO retail method. The following data are available for 2021: CostRetail Beginning inventory$71,280 $132,000 Net purchases 112,500 255,000 Net markups 6,000 Net markdowns 11,000 Net sales 232,000 Retail price index, 12/31/2021 1.04 Required:Calculate the estimated ending inventory and cost of goods sold for 2021 using the information provided. (Do not round intermediate calculations.)

Ending inventory at retail; 150,000 ending inventory at cost 77,004 cogs 106,776

Present and future value tables of $1 at 11% are presented below. On October 1, 2021, Justine Company purchased equipment from Napa Inc. in exchange for a noninterest-bearing note payable in five equal annual payments of $500,000, beginning Oct 1, 2022. Similar borrowings have carried an 11% interest rate. The equipment would be recorded at:

Equipment would be recorded at = $500,000 * Cumulative PV factor at 11% for 5 periods Present value = Future value / (1 + i)^n = $500,000 * 3.69590 = $1,847,950

9.Intangible assets usually are reported in the balance sheet as current assets.( true or false)

False

A common output method used to measure progress towards completion is to compare cost incurred to date to total costs estimated to complete the job.

False

A deferred annuity is one in which interest charges are deferred for a stated time period.

False

Which of the following is an example of a variable consideration?

Excellent Electronics has a 10% mail-in rebate program for the Model X-001 speaker system. The company sold $10,000 worth of systems and believes there is a 50% chance that rebates will be redeemed.

Research and development costs for projects other than software development should be:

Expensed in the period incurred.

Securitization of receivables is a type of secured borrowing

FALSE

4b. Statements of Financial Accounting Concepts issued by FASB

FASB Identify the conceptual framework within which accounting standards are developed.

In a period when costs are rising and inventory quantities are stable, the inventory method that would result in the highest ending inventory is:

FIFIO

Donated assets are recorded at:

Fair value

The basic principle used to value an asset acquired in a nonmonetary exchange is to value it at:

Fair value of the asset(s) given up.

10. The statement of shareholders' equity discloses the changes in the temporary shareholders' equity accounts( True or false)

False

11. Subsequent events are significant developments that take place after a firm's year-end, and after the financial statements are issued or available to be issued.( true or false)

False

12.All illegal acts should be disclosed in the notes to the financial statements.( true or false)

False

annuity due

First cash flow occurs on the first day of the agreement

ordinary annuity

First cash flow occurs one period after agreement begins

A contractual arrangement under which one party grants another party the exclusive right to use a trademark or tradename is a:

Franchise

In applying LCM, market cannot be

Greater than net realizable value.

3. Highsmith Rental Company purchased an apartment building early in 2021. There are 20 apartments in the building and each is furnished with major kitchen appliances. The company has decided to use the group depreciation method for the appliances. The following data are available: ApplianceCostResidual ValueService Life(in Years)Stoves$15,000 $3,000 6Refrigerators 10,000 1,000 5Dishwashers 8,000 500 4 In 2024, three new refrigerators costing $2,700 were purchased for cash. In that same year, the old refrigerators, which originally cost $1,500, were sold for $200.Required:1. Calculate the group depreciation rate, group life, and depreciation for 2021.2. Prepare the journal entries to record the purchase of the new refrigerators and the sale of the old refrigerators.

Group depreciation rate 17.20 Group life 5.02 Depreciation for 2021 (sta8 line) 5,675 Req 2 2700dr Refridegrator 2700cr cash Cash 200dr accumlated depreciation 1300dr refridgetaor 1500cr

Which of the following is one of the steps for recognizing revenue?

Identify the performance obligations of the contract.

Which of the following statements is/are true?

In a period of rising costs and stable inventory levels, using the FIFO method leads to a higher taxable income and higher net income compared to the LIFO method.

When is a loss recognized on a long-term contract?

In the first period in which the loss becomes evident

When changing from the average cost method to FIFO, the company:

Includes in current year's income the cumulative after-tax difference that would have resulted if the company had used FIFO in all prior years. Revises comparative financial statements. Records a journal entry to adjust the book balances from their current amounts to what those balances would have been using FIFO.

A major addition to equipment should have been capitalized in the year 2021 but was incorrectly expensed. Which of the following is (are) true?

Income in 2021 is understated. Income in future years is overstated. Assets in 2021 are understated.

Retained earning example

Income less dividends, accumulated

Comparable/ Comparability

Information is comparable if similar items are treated the same way and different items are treated differently.

11. On December 31, 2017, Coolwear, Inc. had a balance in its prepaid insurance account of $48,400. During 2018, $86,000 was paid for insurance. At the end of 2018, after adjusting entries were recorded, the balance in the prepaid insurance account was 42,000. Insurance expense for 2018 would be

Insurance expense = $48,400 + 86,000 − 42,000 = $92,400

compound interest

Interest calculated on invested amount plus accumulated interest

18.Mama's Pizza Shoppe borrowed $8,000 at 9% interest on May 1, 2018, with principal and interest due on October 31, 2019. The company's fiscal year ends June 30, 2018. What adjusting entry is necessary on June 30, 2018?

Interest expense120dr Interest payable 120cr 8000* 9% * 2/12=$120

Interest is not capitalized for:

Inventories routinely and repetitively produced in large quantities.

Average accumulated expenditures:

Is an approximation of the average debt a firm would have outstanding if it financed all construction through debt.

Zulu Corporation hires a new chief executive officer and promises to pay her a signing bonus of $2 million per year for 10 years, starting five years after she joins the company. The liability for this bonus when the CEO is hired

Is the present value of a deferred annuity

External users of a company's financial information include

Labor unions, banks, credit rating agencies

10.5 Spice World Corporation purchased for $180,000 cash the following assets: land with a fair value of $60,000, a building with a fair value of $90,000, equipment with a fair value of $35,000 and inventory with a fair value of $15,000. Prepare the appropriate journal entry for Spice World's purchase.

Land 54,000dr Building 81,000dr Equipment 31,500dr Inventory 13,500dr Cash 180,00cr ( * everything by 180,000* %) work_ 60,000+90,000+35,000+15,000=200,000 $60,000/200,000 =30% $90,000/200,000 =45% $35,000/200,000=17.5% $15,000/200,000+=7.5%

Which of the following is an example of an extended warranty?

Mark Electronics offers a warranty at an affordable price that provides additional protection after the customer takes possession of the product

Compared to dollar-value LIFO, unit LIFO is:

More costly to implement.

Under the dollar-value LIFO retail method, to determine the value of a LIFO layer:

Multiply the LIFO layer by the layer-year price index and by the layer-year cost-to-retail percentage

Long term liabilities

Not payable, due in 5 years

Data related to the inventories of Costco Medical Supply are presented below: Surgical Surgical Rehab Rehab Equipment Supplies Equipment Supplies Selling price$260 $100 $340 $165 Cost 170 90 250 162 Costs to sell 30 15 25 10 In applying the lower of cost or net realizable value rule, the inventory of rehab supplies would be valued at:

NRV = $165 - 10 = $155 answ $155 net realizable value is less than $162 cost.

Data related to the inventories of Alpine Ski Equipment and Supplies is presented below: Skis Boots Apparel Supplies Selling price$180,000 $140,000 $120,000 $60,000 Cost 128,000 133,000 90,000 45,000 Replacement cost 120,000 130,000 110,000 41,000 Sales commission 10% 10% 10% 10% In applying the lower of cost or net realizable value rule, the inventory of skis would be valued at:

NRV = $180,000 - ($180,000 × 10%) = $162,000 $128,000 is the answer cost is less than $162,000 net realizable value.

Under the LIFO retail method, the denominator in the cost-to-retail percentage includes:

Net markups and net markdowns.

When computing the cost-to-retail percentage for the average cost retail method, included in the denominator are:

Net markups and net markdowns.

Under the conventional retail method, the denominator in the cost-to-retail percentage includes:

Net markups, but not net markdowns.

Neutral/ neutrality

Neutral/ neutrality The FASB faces a difficult task in maintaining neutrality in the faces of economic consequences and resulting political pressures

nonmonetary asset

No fixed dollar amount attached

Monetary liability

Obligation to pay a sum of cash, the amount of which is fixed

Working capital

Occurs after the fiscal year-end but before the statements are issued

In a periodic inventory system, the cost of purchases is debited to:

PURCHASES

5 (17.) An investment product promises to pay $42,000 at the end of 10 years. If an investor feels this investment should produce a rate of return of 12%, compounded annually, what's the most the investor should be willing to pay for the investment?)

PV=FV* (1/1+r)^n $42,000*(1/1+0.12)^10= 13,523

An exclusive 20-year right to manufacture a product or use a process is a:

Patent

Operating cycle

Period of time from payment of cash to collection of cash.

Platter Inc. purchases goods from a supplier and later returns those goods. Platter uses a perpetual system. Which of the following statements is correct?

Platter will record the return by reducing its Inventory account.

(5.1) For each of the following situations involving single amounts, solve for the unknown. Assume that interest is compounded annually. (i = interest rate, and n = number of years) (FV of $1, PV of $1, FVA of $1, PVA of $1, FVAD of $1 and PVAD of $1) (Use appropriate factor(s) from the tables provided. Round your final answers to nearest whole dollar amount.)

Present value = Future value / (1 + i)^n Interest rate = (Future value / Present value)^(1/n) - 1 Future value = Present value (1 + i)^n Interest rate = (Future value / Present value)^(1/n) - 1 Future value = Present value (1 + i)^n/i-1 PV FV I N 1. 24,837 $40,000 10% 5 2. 36,289 $65,000 6% 10 3. 15,884 $40,000 8% 12 4. 46,651 $100,000 10% 8 5, 15,376 $59,500 7% 20

5 (18.)An uncle asks to borrow $1,000 today and promises to repay you $1,210 two years from now. To find the annual interest rate you would be agreeing to, you would search the second row in the: (PV of $1

Present value of $1 table, for the factor closet to 0.82645 1,000/1,210

(5 .15.) Today, Thomas deposited $100,000 in a three-year, 12% CD that compounds quarterly. What is the maturity value of the CD?

Present value=100,000 12/4= 3% 0r .03 number of periods 3*4= 12 ANSW= 100,000(1+.03)^12

15. The distinction between operating and nonoperating income relates to:

Primary activities of the reporting activities

The income statement reports a company's profit during a particular reporting period.

Profit= (revenues & gains - expenses & losses)

4.) Harwell Company manufactures automobile tires. On July 15, 2018, the company sold 1,000 tires to the Nixon Car Company for $50 each. The terms of the sale were 2/10, n/30. Harwell uses the gross method of accounting for cash discounts ( gross method). Required1. Prepare the journal entries to record the sale on July 15 (ignore cost of goods) and collection on July 23, 2018.2.2.Prepare the journal entries to record the sale on July 15 (ignore cost of goods) and collection on August 15, 2018.

Re 1 July 15 ( account receivable 50,00 dr, sale revenue 50,00 cr) July 23 ( cash =50,000 98%= 49,000 dr, Sales discount $50,0002%=1,000, account receivable 50,000 ) Req 2 July 15 (Account receovable 50,000 dr, sale revenue 50,000 cr AUG 15 cash 50,00dr, account receivable 50,000 cr

5.) Harwell Company manufactures automobile tires. On July 15, 2018, the company sold 1,000 tires to the Nixon Car Company for $50 each. The terms of the sale were 2/10, n/30. Harwell uses the net method of accounting for cash discounts. Required:1. Prepare the journal entries to record the sale on July 15 (ignore cost of goods) and payment on July 23, 2018.2. Prepare the journal entries to record the sale on July 15 (ignore cost of goods) and payment on August 15, 2018.

Re 1 50,000* 98%= 49,000 July 15 account receivable 49,00 dr, sale revenue 49,00 cr July 23 cash 50,000 dr, account receivable 49,000 )= Req 2 July 15 Account receovable 49,000 dr, sale revenue 49,000 cr AUG 15 cash 50,00dr, account receivable 50,000 cr, sales forfited 50,00-49,000= 1,000 cr

Which one of the following is not one of the five steps for recognizing revenue?

Recognize revenue when all the performance obligations have been satisfied

4d. In depreciating the cost of an asset, accountants are most concerned with

Recognizing expense in the appropriate period.

the primary objective of the matching principle is to

Record expenses in the period that related revenues are recognized

Accrued liabilities

Recorded when an expense is incurred but not yet paid.

An impairment loss has the effect of:

Reducing total assets

Revenue recognition .

Refers to the process of admitting information into the financial statement criteria are usually satisfied for products at point of sale

Madison Co. has determined its year-end inventory on a LIFO basis to be $600,000. Information pertaining to that inventory is as follows: Selling price$720,000 Costs to sell 30,000 Normal profit margin 80,000 Replacement cost 620,000 What should be the reported value of Madison's inventory?

Replacements cost 620,000 basis is 600,000 720,000-30,000-80,000= 610,000 cost is less than 620,000 ANSW= 600,000

10. The Claxton Company manufactures children's toys and also has a division that makes automobile parts. Due to a change in its strategic focus, the company sold the automobile parts division. The division qualifies as a component of the entity according to GAAP. How should Claxton report the sale in its 2018 income statement?

Report the income or loss from operations of the division in discontinued operations.

3.)Tracy Company, a manufacturer of air conditioners, sold 100 units to Thomas Company on November 17, 2018. The units have a list price of $600 each, but Thomas was given a 30% trade discount. The terms of the sale were 2/10, n/30. Required:1. Prepare the journal entries to record the sale on November 17 (ignore cost of goods) and collection on November 26, 2018, assuming that the gross method of accounting for cash discounts is used. 2. Prepare the journal entries to record the sale on November 17 (ignore cost of goods) and collection on December 15, 2018, assuming that the gross method of accounting for cash discounts is used .3-a. Prepare the journal entries to record the sale on November 17 (ignore cost of goods) and collection on November 26, 2018, assuming that the net method of accounting for cash discounts is used .3-b. Prepare the journal entries to record the sale on November 17 (ignore cost of goods) and collection on December 15, 2018, assuming that the net method of accounting for cash discounts is used.

Req 1 NOV 17 Account receivable 42,000dr Sale revenue 42,000cr $600 = $60,000 × 70% = $42,000 Nov 26 Cash: (98% × $42,000) = $41,160dr Sales discounts: (2% × $42,000) = $840dr Account receivable 42,000 DR Req2 Account receivable 42,000dr Sale revenue 42,000cr $600 = $60,000 × 70% = $42,000 Nov 26 Cash: (98% × $42,000) = $42,000dr Account receivable 42,000 DR 3A.) Nov 17 account receivable 41,160 dr, Sale revenue ) 41,160 cr) Nov 26 cash 41, 160 dr, account receivable 41,160 cr 3B.) Nov 17 ( account receivable 41,160 dr, sale revenue 41,160 cr) Dec 15 Cash 42,000 , Account receivables 41,160 , sale discount foveated 840) 98%*42,000= 41,160 2%*42000=840

54.) Jing Statistical Services operates a website that links experienced statisticians with businesses that need data analyzed. Statisticians post their rates, qualifications, and references on the website, and Jing receives 25% of the fee paid to the statisticians in exchange for identifying potential customers. VetMed Associates contacts Jing and arranges to pay a consultant $1,500 in exchange for analyzing some data. Jing's income statement would include the following with respect to this transaction:

Revenue of $375= (computed as 25% × $1,500).

On December 15, Reynolds Inc. ships goods on consignment to Manella Inc., who receives the goods on December 20. Manella sells the goods to a customer on January 5 of the following year. Both companies have December 31 year-ends. Which of the following statements is correct?

Reynolds will include the goods in its December 31 inventory.

Abbott Company uses a perpetual inventory system. When does Abbott record transactions relating to its inventory?

Sales revenue and cost of goods sold are both recorded at the time of the sale.

136,400/ 1.10=124,000124,000-112,000=12,00012,000 *1.10=13,200112,000+13,200=125,200

TRUE

Net purchases are reduced for discounts taken whether the net method is used or the gross method is used.

TRUE

The primary reason most companies choose LIFO is to reduce tax payments.

TRUE

The receivables turnover ratio provides a way for an analyst to assess the effectiveness of a company in managing its investment in receivables.

TRUE

If Nixon instead uses a periodic inventory system, explain any changes to the journal entries created in requirements 1 and 2.

The July 15 entry would include a debit to the purchase account instead of to inventory and the July 23entry would include a credit to the Purchase discounts account instead of to Inventory

present value of a single amount

The amount of money that a dollar will grow to

The economic entity assumption

The economic entity assumption presumes that economic events can be identified with a particular economic entity.

Assets acquired by the issuance of equity securities are valued based on:

The fair value of the assets acquired or the fair value of the equity securities, whichever is more reasonably determinable.

In a nonmonetary exchange of equipment, if the exchange has commercial substance, a gain is recognized if:

The fair value of the equipment surrendered exceeds the book value of the equipment given up.

Interest is eligible to be capitalized as part of an asset's cost, rather than being expensed immediately, when:

The interest is incurred during the construction period of the asset. The asset is a discrete construction project for sale or lease. The asset is self-constructed, rather than acquired.

On June 17, the Lattern Company issued 120,000 shares of its $0.10 par value common stock in exchange for land. On the date of the transaction, the fair value of the common stock, evidenced by its market price, was $10 per share. The journal entry to record this transaction includes:

The land is recorded at the fair value of the stock exchanged = 120,000 shares × $10 per share = $1,200,000. Debited Land, 1,200,000

The key distinction between current liabilities and long-term liabilities is:

The length of time until the obligation is expected to be satisfied—less than one year versus more than one year, or operating cycle if longer

An asset acquired using a long-term note payable always will be recorded at the face amount of the note under which scenario?

The note payable explicitly requires the payment of interest at a realistic interest rate.

Historical cost principle

The original transaction value upon acquisition.

effective rate or yield

The rate at which money will actually grow during a year

22. Liquidity refers to

The readiness of an asset to be converted to cash

10. The criteria for determining which items comprise cash equivalents often is disclosed in the summary of significant accounting policies( True or false)

True

2. Debits increase asset accounts and decrease liability accounts.( True or false)

True

4. A change in accounting principle that is implemented using the retrospective approach includes restating financial statements of all periods presented as if the new standard had been used in those periods (true or false)

True

50.) On April 1st, Bob the Builder entered into a contract of one-month duration to build a barn for Nolan. Bob is guaranteed to receive a base fee of $5,000 for his services in addition to a bonus depending on when the project is completed. Nolan created incentives for Bob to finish the barn as soon as he can without jeopardizing the structural integrity of the barn. Nolan offered to pay an additional 30% of the base fee if the project finished 2 weeks early and 10% if the project finished a week early. The probability of finishing 2 weeks early is 30% and the probability of finishing a week early is 60%.What is the expected transaction price with variable consideration estimated as the most likely amount?

The total expected amount is $5,000 + ($5,000 × 10%) = $5,500

Which of the following statements are true regarding potential impairment of intangible assets with indefinite lives

This type of asset should be tested for impairment whenever events or changes in circumstances indicate that it is more likely than not that the asset is impaired. This type of asset should be tested for impairment whenever events or changes in circumstances indicate that it is more likely than not that the asset is impaired. If book value exceeds fair value, an impairment loss is recognized for the difference.

Gunk Goblin sells vacuums and just launched a policy where customers have the right to return a vacuum during a three-year period following purchase. Gunk management has no experience under this sort of policy and does not believe it can accurately estimate returns. What is the longest period of time that Gunk may have to wait before recognizing revenue associated with one of these sales?

Three years, after the right of return has expired.

The exclusive right to display a symbol of product identification is a:

Trademark

In a consignment arrangement, revenue typically should not be recognized until sale to a third party occurs, even though there has been a physical transfer of goods to the consignee, because the consignor still retains legal title to the goods.

True

In the future value of an ordinary annuity, the last cash payment will not earn any interest

True

Materiality can be affected by the dollar amount of an item, the nature of the item, or both. True or False

True

Most, but not all, liabilities are monetary liabilities

True

Net realizable value is selling price less costs of completion, disposal, and transportation.

True

Property, plant, and equipment and intangible assets are long-term, revenue producing assets.

True

Revenue should be recognized over time for the construction of an annex to a building that the customer owns, even if the seller will not receive payment until the annex is completed.

True

Revenue typically should not be recognized when payment is received but the goods are warehoused at the seller's facility.

True

The FASB's framework for measuring fair value doesn't change the situations in which fair value is used under current GAAP. True or flase

True

The FASB's required accounting treatment for research and development costs often understates both net income and assets.

True

The Public Company Accounting Reform and Investor Protection Act of 2002 (Sarbanes-Oxley) changed the entity responsible for setting auditing standards in the United States. True or False

True

The capitalized cost of cloud computing arrangements is amortized over the software's expected useful life.

True

The fair value of the asset, debt, or equity securities given in a noncash acquisition should determine the value of the consideration received.

True

The monetary unit assumption requires that items in financial statements be measured in a particular monetary unit. True or false

True

The primary function of financial accounting is to provide relevant financial information to parties external to business enterprises. True or False

True

The primary motivation behind the lower of cost or market (LCM) rule is conservatism

True

The purpose of the conceptual framework is to provide a structure and framework for a consistent set of GAAP. True or False

True

The relative fair values are used to determine the valuation of individual assets acquired in a lump-sum purchase

True

The revenue/expense approach emphasizes determining the appropriate amounts of revenue and expense in each reporting period. (true or false)

True

The three factors in cost allocation of a depreciable asset are service life, allocation base, and allocation method.

True

The transaction price is only allocated to goods or services that are both capable of being distinct and that are separately identifiable.

True

The transaction price should be allocated to the contract's performance obligations in proportion to the stand-alone selling prices of the performance obligations.

True

Under federal securities laws, the SEC has the authority to set accounting standards in the United States. True or False

True

Unit LIFO is more costly to implement than dollar-value LIFO

True

Using the balance sheet approach, bad debt expense is an indirect result of estimating the appropriate balance for the allowance for uncollectible accounts.

True

When recognizing revenue over time on a long-term contract, the percent complete is often estimated by comparing the cost incurred to date with the total estimated cost to complete.

True

With an annuity due, a payment is made or received on the date the agreement begins.

True

9.) Jameson Company uses average cost and a perpetual system. On January 1, the company had 600 units of inventory at an average cost of $55 per unit for a total cost of $33,000. The company purchased and sold inventory during the month as follows: Purchases:January 10: 1,000 units at $59 = $59,000January 20: 800 units at $62 = $49,600 Sales:January 12: 1,200 unitsJanuary 28: 900 units What is the average cost per unit that should be used to determine the cost of the units sold on January 28? (Round your answer to two decimal points.)avg cost per unit

Units available for sale = 600 + 1,000 = 1,600 Average cost per unit = $92,000 ÷ 1,600 units = $57.50 At January 12: 1,600 units available − 1,200 units sold = 400 unitsCost of units available for sale = 400 × $57.50 = $23,000 At January 20:Cost of units available for sale = $23,000 + $49,600 = $72,600 Units available for sale = 400 + 800 = 1,200 Average cost per unit = $72,600 ÷ 1,200 units = $60.50

Which of the following statement is most true?

Variable consideration means that the transaction price is uncertain.

Boomerang Computer Company sells computers with an unconditional right to return the computer if the customer is not satisfied. Boomerang has a long history selling these computers under this returns policy and can provide precise estimates of the amount of returns associated with each sale. Boomerang most likely should recognize revenue:

When Boomerang delivers a computer to a customer, in an amount that is reduced by the expected returns.

Current liabilities

Will be satisfied through the use of current assets

18. Current assets include cash and all other assets expected to become cash or be consumed:

Within one year or one operating cycle, whichever is longer.

24.) Fulbright Corp. uses the periodic inventory system. During its first year of operations, Fulbright made the following purchases (listed in chronological order of acquisition): 40 units at $100 70 units at $80 170 units at $60 Sales for the year totaled 270 units, leaving 10 units on hand at the end of the year.

[(40 × $100) + (70 × $80) + (170 × $60)] = $19,800 ÷ 280 units = $70.71 per unit 10 units × $70.71 = $707 (rounded)

Which of the following is not a performance obligation?- A good that the seller could sell separately and that is separately identifiable from other goods or services in the contract.- A right of return.Correct- An option for a customer to purchase goods under terms that are more advantageous than those enjoyed by other customers.-An extended warranty.

a right of return

Corning Industries owns a patent for which it paid $77,000. At the end of the current year, accumulated amortization on the patent totaled $14,000. Due to adverse economic conditions, Corning's management determined that it should assess whether an impairment loss should be recognized for the patent. The estimated undiscounted future cash flows to be provided by the patent total $45,000, and the patent's fair value is $30,000. (a) What is the amount of the impairment loss, if any, on the patent at the end of the current year? (b) What is the book value of the patent after any impairment loss is recorded?

a. impairment cost (77,000-14,000)= 63,000-30,000= 33,000 b. book value 30,000 fair value

Current liabilities example

account payable, deferred rent revenue for the next 12 months, note payable, due in 6 months, accrued liabilities, taxes payable

On November 10 of the current year, Flores Mills sold carpet to a customer for $8,000 with credit terms 2/10, n/30. Flores uses the gross method of accounting for cash discounts.What is the correct entry for Flores on November 10?

account receivable 8000dr sales 8000 cr

Accrual-basis

accounting doesn't focus only on cash flows. Instead, it also reflects other resources provided and consumed by operations during a period. The accrual accounting model's measure of resources provided by business operations is called revenues, and the measure of resources sacrificed to produce revenues is called expenses

Cash basis

accounting produces a measure called net operating cash flow. This measure is the difference between cash receipts and cash payments from transactions related to providing goods and services to customers during a reporting period.

Adding a refrigeration unit to a delivery truck that previously did not have this capability is an example of:

addition

The periodicity assumption .

allows the life of a company to be divided into artificial time periods to provide timely information

9.8 Data related to the inventories of Costco Medical Supply are presented below: Surgical Surgical /Rehab Rehab /Equipment Supplies/ Equipment Supplies Selling price$260 $100 $340 $165 Cost 170 90 250 162 Costs to sell 30 15 25 10 In applying the lower of cost or net realizable value rule, the inventory of surgical equipment would be valued at:

answ. 170 $260 - 30 = $230$170 cost is less than the $230 net realizable value.

The primary responsibility for properly applying GAAP when communicating with investors and creditors through financial statements lies with a firm's auditors. True or False

false

On November 10 of the current year, Flores Mills sold carpet to a customer for $8,000 with credit terms 2/10, n/30. Flores uses the gross method of accounting for cash discounts.What is the correct entry for Flores on December 5, assuming the correct payment was received on that date

cash 8000 dr account receivable 8000 cr

chapter 6

chapter 6

chapter 7

chapter 7

37.) In addition, you determine that the June 30, 2021, inventory balance is $40,000.Required:Calculate the cost of goods sold for the Askew Company for the year ending June 30, 2021

chapter 8 notes

38.) Applying both a perpetual and a periodic inventory system, prepare the journal entries that summarize the transactions that created these balances. Include all end-of-period adjusting entries indicated.

chapter 8 notes

39,)Prepare the journal entries to record the purchase on July 15 and payment on July 23, 2021.2. Prepare the journal entry for the payment, assuming instead that it was made on August 15, 2021.

chapter 8 notes

chapter 9

chapter 9

Herman Company has three products in its ending inventory. Specific per unit data at the end of the year for each of the products are as follows: Product 1 Product 2 Product 3Cost$20 $90 $50 Replacement cost 18 85 40 Selling price 40 120 70 Selling costs 6 40 10 Normal profit 5 30 12 Required:What unit values should Herman use for each of its products when applying the lower of cost or market (LCM) rule to ending inventory?

chapter 9 notes

Herman Company has three products in its ending inventory. Specific per unit data at the end of the year for each of the products are as follows: Product 1 Product 2 Product 3Cost$20 $90 $50 Selling price 40 120 70 Costs to sell 6 40 10 What unit values should Herman use for each of its products when applying the lower of cost or net realizable value (LCNRV) rule to ending inventory?

chapter 9 notes #32

General ledger

collection of accounts

If a contract qualifies for revenue recognition over time, revenue is recognized based on progress toward

completion

The exclusive right to benefit from a creative work, such as a film, is a

copyright

44. On March 1, 2021, Beldon Corporation purchased land as a factory site for $65,000. An old building on the property was demolished, and construction began on a new building that was completed on December 15, 2021. Costs incurred during this period are listed below: Demolition of old building $6,500 Architect's fees (for new building) 17,000 Legal fees for title investigation of land 4,500 Property taxes on land (for period beginning March 1, 2021) 3,500 Construction costs 550,000 Interest on construction loan 7,500 Salvaged materials resulting from the demolition of the old building were sold for $2,500. Required:Determine the amounts that Beldon should capitalize as the cost of the land and the new building.

cost of land ) Captilized cost of land 65,000 purchase price 6,500 demolition of old building 4,5000 legals fees for title investigation (2,500) Total cost of land 73,5000 Cost of new building) Captalized cost of building Construction cost 550,000 interest on construction loan 7,500 total cost of building 574,5000

20.Red Onion Restaurant would classify a six-month prepaid insurance policy as:

cuilding

19. Red Onion Restaurant would classify a six-month prepaid insurance policy as:

current asset

Accounts represent

elements of the accounting equation.

Property, plant and equipment example

equipment, building in use, land in use

If the contract contains multiple performance obligations, revenue must be recognized in an amount equal to the fair value of each of the separate performance obligations.

false

In a good system of internal control, the person who initiates a transaction should be allowed to effectively control the processing of the transaction through its final inclusion in the accounting records

false

In the United States, the conceptual framework indicates GAAP when a more specific accounting standard does not apply. True or False

false

LIFO periodic and LIFO perpetual always produce the same dollar amounts for ending inventory.

false

Unqualified opinion

he statements are presented fairly in conformity with GAAP

4.Chadwick Enterprises, Inc., operates several restaurants throughout the Midwest. Three of its restaurants located in the center of a large urban area have experienced declining profits due to declining population. The company's management has decided to test the assets of the restaurants for possible impairment. The relevant information for these assets is presented below. Book value$6.5million Estimated undiscounted sum of future cash flows 4.0million Fair value 3.5million Required:1. Determine the amount of the impairment loss.2. Determine the amount of the impairment loss assuming that the estimated undiscounted sum of future cash flows is $6.8 million and fair value is $5 million.

impairment loss 6.5-3= 3.5 impairment cost 0

Summary of significant accounting policies

important to a user in comparing financial information across companies

41.) Prepare the journal entries to record the purchase on July 15 and payment on July 23, 2021.2. Prepare the journal entry for the payment, assuming instead that it was made on August 15, 2021.

in chapter 8n notes

29.( Under the dollar-value LIFO method, the inventory at December 31, 2019, should be

in notes

36>) John's Specialty Store uses a perpetual inventory system. The following are some inventory transactions for the month of May: 1.) John's purchased merchandise on account for $5,000. Freight charges of $300 were paid in cash. 2.) John's returned some of the merchandise purchased in The cost of the merchandise was $600 and John's account was credited by the supplier 3.).Merchandise costing $2,800 was sold for $5,200 in cash.

in notes

Mercury Inc. purchased equipment in 2019 at a cost of $400,000. The equipment was expected to produce 700,000 units over the next five years and have a residual value of $50,000. The equipment was sold for $210,000 part way through 2021. Actual production in each year was: 2019 = 100,000 units; 2020 = 160,000 units; 2021 = 80,000 units. Mercury uses units-of-production depreciation, and all depreciation has been recorded through the disposal date. Required:1. Calculate the gain or loss on the sale.2. Prepare the journal entry to record the sale.3. Assuming that the equipment was instead sold for $245,000, calculate the gain or loss on the sale.4. Prepare the journal entry to record the sale in requirement 3.

in notes chapter 11

in 2021, internal auditors discovered that PKE Displays, Inc. had debited an expense account for the $350,000 cost of equipment purchased on January 1, 2018. The equipment's life was expected to be five years with no residual value. Straight-line depreciation is used by PKE. Required:1. Determine the cumulative effect of the error on net income over the three-year period from 2018 through 2020, and on retained earnings by the end of 2020.2. Prepare the correcting entry assuming the error was discovered in 2021 before the adjusting and closing entries. (Ignore income taxes.)3. Assume instead that the equipment was disposed of in 2022 and the original error was discovered in 2023 after the 2022 financial statements were issued. Prepare the correcting entry in 2023.

in notes chapter 11

Using the gross method, purchase discounts lost are:

include in purchases

cost effective

information is cost effective if the benefit of increased decision usefulness exceeds the costs of providing that information.

Longer term assets include

investments, property, plants, and equipment, intangible assets, other long term assets

Accounts receivable

is a permanent asset account. account, so it is increased with a credit.

4a. The primary objective of financial reporting

is to provide information useful to capital providers

Timeliness

is when time is available to users early enough to make a decision, and the SEC requires its resistant to submit finical statement information quarterly as on an annual basis for each fiscal year.

On June 1st, Lucy & Bros received an order for 500 cupcakes. Lucy delivered the cupcakes to the client on June 25th. A $50 deposit was received on June 5th and the remaining $450 was paid on June 30th. Lucy likely would recognize revenue on:

june 25th

If costs are rising, which inventory method will result in the highest cost of goods sold?

lifo

ABC Lawncare performed lawn maintenance services for Drake Inc. on June 1st, and received payment of $500 for those services.II. On June 1st, Melly Corp received payment for 100 pounds of raw material to be delivered to Drake Inc. in 6 months.III. Lodo, LLC collected cash on June 1st for services rendered on May 1st.

ll, lll only

Expense recognition, materiality

materiality are outflows or other using up of assets or incurrences of liabilities from providing goods or services

Faithful representation

means agreement between a measure and a real-world phenomenon that the measure is supposed to represent

9. Most real-world income statements are presented using which format?

multiple-step

Under the conventional retail method, which of the following is not included in the denominator of the cost-to-retail percentage?

net markdowns

2. The application of intraperiod income taxes requires that income taxes be apportioned to each of the following items except:

operating income

13. How are management's responsibility and the auditors' opinion on internal controls represented in the unqualified auditor's report? Management'sResponsibility Auditors'Responsibility a.Not stated Stated b.Stated Stated c.Not stated Not stated d.Stated Not stated

option b

Consistency

the financial statement a summary of significant accounting policies and provide full disclosure of any changes in those policies to alert users to the potential for diminished consistency

On February 1st, H&B Bank originated a loan for $50,000 at an interest rate of 7.2%. On March 15th, an interest payment of $300 was received. Which of the following best describes when interest revenue should be recognized?

overtime

8.) Platen purchased inventory on August 17 and received an invoice with a list price amount of $5,900 and payment terms of 4/10, n/30. Platen uses the net method to record purchases. For what amount should Platen record the purchase?platen record the purchase

platen record the purchase=$ 5,900 − ($5,900 × 4%) = $5,664

economic entity assumption

presumes that economic events can be identified with a particular economic entity.the enterprise is separate from its owners and other entities

The historical cost(original transaction value

principle bases measurements on the amount given or received in the original exchange transaction

12. When a material error is discovered in prior financial statements:

prior financial statements are restated to their correct amounts. assets and liabilities in the current period are restated to their appropriate levels. prior income effects are adjusted to the current period's beginning balance of retained earnings.

16. Chance Company had two operating divisions, one manufacturing farm equipment and the other office supplies. Both divisions are considered separate components as defined by generally accepted accounting principles. The farm equipment component had been unprofitable, and on September 1, 2018, the company adopted a plan to sell the assets of the division. The actual sale was completed on December 15, 2018, at a price of $600,000. The book value of the division's assets was $1,000,000, resulting in a before-tax loss of $400,000 on the sale. The division incurred a before-tax operating loss from operations of $130,000 from the beginning of the year through December 15. The income tax rate is 40%. Chance's after-tax income from its continuing operations is $350,000. in nRequired:Prepare an income statement for 2018 beginning with income from continuing operations. Include appropriate EPS disclosures assuming that 100,000 shares of common stock were outstanding throughout the year.

put in notes

Expense

recognition Record expenses in the period the related revenue is recognized.

Double-entry system

refers to the dual effect that each transaction has on the accounting equation

monetary unit assumption

states that financial statement elements should be measured in a particular monetary unit (in the United States, the U.S. dollar). Monetary unit assumption A common denominator is the dollar.

Periodicity

the life of an enterprise can be divided into artificial time periods.

Materiality Concerns

the relative size of an item and its effect on decision

Liquidity Refers

to the ability of a company to convert its assets to cash to pay its current obligations.

1. Owners' equity can be expressed as assets minus liabilities.( True or False)

true

8. The multiple-step format of the income statement reports a series of intermediate subtotals such as gross profit, operating income, and income before taxes.( True or false)

true

A transfer of goods or services is complete when the customer has control over the goods or services.

true

An annuity is a series of equal periodic payments.

true

Companies recognize revenue when goods or services are transferred to customers for the amount the company expects to be entitled to receive in exchange for those goods or services

true

Depending on the circumstances, the classification of a compensating balance may be either current or noncurrent, and the arrangement should be disclosed in the notes

true

Equipment that will be used to extract natural resources should be depreciated over its useful life or the life of the natural resource, whichever is shorter, assuming the equipment has no alternative use.

true

For intangible assets used in the manufacture of a product, amortization is a product cost and is included in the cost of inventory.

true

For intangible assets with indefinite useful lives, estimated future cash flows generally are not used as part of the recoverability test

true

From a financial accounting perspective, the main purposes of a system of internal control are to improve the accuracy and reliability of accounting information and to safeguard assets.

true

Generally accepted accounting principles allow a company to change from one depreciation method to another if the company can justify the change.

true

The FASB's due process invites various interested parties to indicate their opinions about whether financial accounting standards should be changed

true

The interest capitalization period for a self-constructed asset ends either when the asset is substantially complete and ready for use or when interest costs no longer are being incurred

true

15.Yummy Foods purchased a two-year fire and extended coverage insurance policy on August 1, 2018, and charged the $4,200 premium to Insurance expense. At its December 31, 2018, year-end, Yummy Foods would record which of the following adjusting entries?

unused 12/31= 4200 *19/24= $3,325 Prepaid insurance3,325dr Insurance expense 3,325cr


संबंधित स्टडी सेट्स

BJU Press US History Ch. 13 section quizzes

View Set

Present tense of irregular yo verbs

View Set

EXAM #2 Practice NCLEX Questions

View Set

FIN 320 chap 4 Time Value of Money: Valuing Cash Flow Streams

View Set